展开全部

题目列表

题目内容
The author would likely agree with which of the following statements about the “black holes [the LHC] could conceivably form"?
The author presents the “potential caveat” as
Which of the following statements best describes the function of the first paragraph in the context of the passage as a whole?
The author suggests that the argument advanced in the second paragraph of the passage may be challenged for which of the following reasons?
Which of the following most logically completes the argument?
The author of the passage makes which of the following claims about the "theorists"?
The passage suggests that one reason the United States women's movement became more involved in electoral politics than did the Canadian women's movement was that
Which of the following can be inferred from the passage about attempts by some in the women's movement to establish alternative routes for political engagement?
If Laplace's nebular theory were correct, it can most reasonably be assumed that
The author advances the main argument of the passage by
The primary purpose of the passage is to
Which of the following can be inferred from the passage about the places mentioned in the highlighted section?
If the "deposits" were not discovered prior to 150 years ago, then which of the following would be a logically defensible inference?
Which of the following, if true, most seriously weakens the argument?
The highlighted sentence serves primarily to
The passage suggests which of the following about the "reef monitoring surveys”?
The author mentions "Jeffrey Masten" primarily to
According to the passage, the "model of the author"
The author mentions a "border, fence, or corral" primarily to
In the context in which it appears, the word "holds" most nearly means

共收录:

25000 +道题目

201本备考书籍

最新提问